Difference between revisions of "2020 AMC 12A Problems/Problem 17"
Lopkiloinm (talk | contribs) (→Solution 1) |
Lopkiloinm (talk | contribs) (→Solution 1) |
||
Line 6: | Line 6: | ||
==Solution 1== | ==Solution 1== | ||
− | Realize that <math>\ln\frac{91}{90}</math> is extremely small, so the <math>x</math>-coordinates must be very close to each other. Also realize that <math>\frac{91}{90} = \frac{182}{180} = \frac{13*14}{12*15}.</math> <math>\boxed{\textbf{(D) } 12}</math> is the smallest number in that fraction, and therefore must be the answer. ~lopkiloinm | + | Realize that <math>\ln\frac{91}{90}</math> is extremely small, so the <math>x</math>-coordinates must be very close to each other. Because of how small <math>\ln\frac{91}{90}</math> is, we must say that the <math>x</math>-coordinates must be consecutive. Also realize that <math>\frac{91}{90} = \frac{182}{180} = \frac{13*14}{12*15}.</math> <math>\boxed{\textbf{(D) } 12}</math> is the smallest number in that fraction, and therefore must be the answer. ~lopkiloinm |
Revision as of 20:15, 1 February 2020
Problem 17
The vertices of a quadrilateral lie on the graph of , and the -coordinates of these vertices are consecutive positive integers. The area of the quadrilateral is . What is the -coordinate of the leftmost vertex?
Solution 1
Realize that is extremely small, so the -coordinates must be very close to each other. Because of how small is, we must say that the -coordinates must be consecutive. Also realize that is the smallest number in that fraction, and therefore must be the answer. ~lopkiloinm